優化

比較靜態的證明導致最大化問題

  • December 27, 2021

我正在考慮以下問題。

讓 $ f(x,\theta) $ 是一個嚴格的凹實值函式 $ x $ 和 $ c(x) $ 是一個嚴格凸的實值函式 $ x $ , 兩個都 $ x $ 和 $ \theta $ 屬於一個閉區間 $ \mathbb R $ . 此外, $ f $ 和 $ c $ 是二次可微的並且 $ \frac{\partial f}{\partial x\partial \theta}>0 $

讓$$ x^{*}(\theta)=argmax_{x},,f(x,\theta)-c(x) $$

如果 $ x^{}(\theta) $ 是一個內部解決方案,確實 $ x^{} $ 增加 $ \theta $ ?

我對此的回答是肯定的,這就是我證明它的方法。

通過 FOC,我們知道在最優情況下,我們必須有$$ \frac{\partial f(x^{}(\theta),\theta)}{\partial x}-\frac{\partial c(x^{}(\theta))}{\partial x}=0\tag{1} $$

現在如果 $ \theta $ 增加到 $ \theta’ $ , 自從 $ \frac{\partial f}{\partial x\partial \theta}>0 $ , 我們知道$$ \frac{\partial f(x^{}(\theta),\theta’)}{\partial x}>\frac{\partial f(x^{}(\theta),\theta)}{\partial x} $$

因此,我們知道$$ \frac{\partial f(x^{}(\theta),\theta’)}{\partial x}-\frac{\partial c(x^{}(\theta))}{\partial x}>0\tag{2} $$自從 $ x^{}(\theta’) $ 是新的最大化器,我們知道 FOC 仍然保持在 $ x^{}(\theta’) $ , IE$$ \frac{\partial f(x^{}(\theta’),\theta’)}{\partial x}-\frac{\partial c(x^{}(\theta’))}{\partial x}=0\tag{3} $$

比較方程 $ (2) $ 和 $ (3) $ ,並結合以下事實 $ f $ 是嚴格凹的並且 $ c $ 是嚴格凸的 $ x $ , 我們知道 $ x^{}(\theta’)>x^{}(\theta) $ . 因此我們知道 $ x^{*} $ 嚴格增加 $ \theta $ .

我的證明正確嗎?這些問題有什麼定理嗎?

提前致謝!

導出這種比較靜態的教科書方法是使用隱函式定理。

一階條件給出: $$ \dfrac{\partial f}{\partial x}(x^\ast(\theta), \theta) - \dfrac{\partial c}{\partial x}(x^\ast(\theta)) = 0. $$ 區別於 $ \theta $ 給出: $$ \dfrac{\partial^2 f}{\partial x^2} (x^\ast(\theta), \theta) \dfrac{d x^\ast(\theta)}{d \theta} + \dfrac{\partial^2 f}{\partial \theta \partial x}(x^\ast(\theta), \theta) - \dfrac{\partial^2 c}{\partial x^2}(x^\ast(\theta),\theta) \dfrac{d x^\ast(\theta)}{d \theta} = 0 $$ 所以: $$ \dfrac{d x^\ast(\theta)}{d \theta} = \dfrac{-\dfrac{\partial^2 f}{\partial \theta \partial x}(x^\ast(\theta), \theta)}{\dfrac{\partial^2 f}{\partial x^2}(x^\ast(\theta), \theta)- \dfrac{\partial^2 c}{\partial x^2}(x^\ast(\theta))} $$ 如果目標函式是嚴格凹的,那麼分母就是負數,所以 $ \dfrac{d x^\ast(\theta)}{d \theta} $ 如果: $$ \dfrac{\partial^2 f}{\partial \theta \partial x}(x^\ast(\theta), \theta) > 0. $$

引用自:https://economics.stackexchange.com/questions/49856